Maximum Likelihood Estimator Question

Click For Summary
The discussion revolves around finding the Maximum Likelihood Estimator (MLE) for N, the total number of lots in a bag, after drawing two specific lots, X_1 = 17 and X_2 = 30. Participants agree that the problem involves a discrete uniform distribution and that the likelihood function can be expressed based on the probabilities of drawing the two lots without replacement. The key insight is that the likelihood function decreases as N increases, suggesting that the minimum permissible value of N, which is 30, maximizes the likelihood. This leads to the conclusion that the MLE of N is 30, as any larger value would yield a lower likelihood. The discussion highlights the importance of understanding the relationship between the drawn lots and the total number of lots in the bag.
Pyroadept
Messages
82
Reaction score
0

Homework Statement


A bag contains sequentially numbered lots (1,2...N). Lots are drawn at random (each lot has the same probability of being drawn). Two lots are drawn without replacement and are observed to be X_1 = 17 and X_2 = 30. What is the MLE of N, the number of lots in a bag?


Homework Equations





The Attempt at a Solution


Hi everyone,
Here's what I've done so far. I know it has to be the discrete uniform distribution but I'm really very stuck as to how to insert the numbers on the lots into the equation. I can't seem to find any examples like the above question.


--

Use the discrete uniform distribution.
The lots are drawn without replacement, so:

P(X_1) = 1/N
P(X_2) = 1/(N-1)

There are N(N-1) possible combinations of two lots we can draw from the bag.

Hence L(N;X_i) = [(1/N)(1/(N-1))]^((N)(N-1))

l = ln L = N(N-1).ln[1/(N(N-1))]

∂l/∂N = ln[1/(N(N-1))] + (N(N-1))^2

At max, ∂l/∂N = 0

i.e. -ln[1/(N(N-1))] = (N(N-1))^2


---
But here I have two problems in that the equation above is pretty horrible to be working out and also I haven't used the 30 and 17 anywhere in the equation. I know I must be wrong, but I don't know how else I can phrase the answer.

Thanks in advance for any help!
 
Physics news on Phys.org
check this out
http://en.wikipedia.org/wiki/Maximum_likelihood#Examples

i think this is a bit of a trick question to get you thinking... say you know N, the probability of choosing the 2 numbers you got is
P(X_1).P(X_2) = (1/(N))(1/(N-1))

now clearly this is decreasing function of N, so will be maximised by the least value of N alloeable, in this case 30. Any higher value of N would give a lower likelihood
 
Question: A clock's minute hand has length 4 and its hour hand has length 3. What is the distance between the tips at the moment when it is increasing most rapidly?(Putnam Exam Question) Answer: Making assumption that both the hands moves at constant angular velocities, the answer is ## \sqrt{7} .## But don't you think this assumption is somewhat doubtful and wrong?

Similar threads

  • · Replies 6 ·
Replies
6
Views
2K
Replies
10
Views
3K
  • · Replies 2 ·
Replies
2
Views
2K
  • · Replies 7 ·
Replies
7
Views
2K
  • · Replies 9 ·
Replies
9
Views
1K
  • · Replies 1 ·
Replies
1
Views
3K
Replies
10
Views
2K
  • · Replies 3 ·
Replies
3
Views
2K
  • · Replies 4 ·
Replies
4
Views
2K
  • · Replies 4 ·
Replies
4
Views
3K